REG MCQ

अब Quizwiz के साथ अपने होमवर्क और परीक्षाओं को एस करें!

lifetime learning credit is available on a per-taxpayer basis

1. credit of 20% of up to $10000 of tuition and fees paid by a taxpayer for one or more students for graduate and undergraduate courses at an eligible educational institution 2. the credit may be claimed for an unlimited number of years 3. cover tuition paid for the taxpayer, spouse and dependents.

no compete agreement includes

1. necessary to protect the employer's legitimate interests to be enforceable 2. area covered by the agreement must be reasonable 3. the court will consider the ex-employee's ability to find job against the ex-employer's right to protect its business

a corporation's maximum deduction is limited to

10% of AGI

business charitable contribution deduction is limited to

10% of AGI

long term capital gain rate is

15% if you make more than $457600, you pay 20% on long term capital gain

the formula for QBI phase in

20%- ((excess of taxable income over 157k (315k if MFJ)*0.20)) /(50k(100K if MFJ)

Jennie and Steve have one daughter, Kaitlin, who is 21 years old. Kaitlin is a full time student except in the summer when she works as a therapy assistant. She earned $74,500 this summer. Jennie and Steve provide 40% of Kaitlin's support for the year. 20% of her support comes from her grandparents, and Kaitlin provides for the remainder of her support.

Kaitlin meets the age test for a qualifying child bc she is a full time student under age 24. she also does not provide more than 50% of her own support, so she meets the qualifying child requirements.

Deductible taxes include: State and local real property taxes State and local personal property taxes State and local income taxes or sales/use tax (the greater of the two) Foreign income taxes

Nondeductible taxes include: Federal income taxes FICA taxes paid by employees Estate, inheritance, and gift taxes Federal, state, local excise taxes Foreign real property taxes Deductible personal taxes cannot exceed *$10,000* beginning in 2018.

Loss on sale of residence.

Not deductible.

a corporation may deduct up to $5000 of organizational expenditures for the tax year in which the corporation begins business.

The $5000 amount must be reduced by the amount by which organizational expenditures exceed $50000 the remaining expenditures are deducted ratably over the 180 month period beginning with the month the corporation begins business. e.g if expenditure is 5000 , then the amount of deduction for 2018 is 5000+(45000*12/180)=8000

This answer is correct because the Internal Revenue Code provisions dealing with tax return preparation apply to preparers. A preparer is defined as an individual or firm who prepares returns for compensation.

The compensation can be implied or explicit. This is an example of implied compensation. Apply to a CPA who prepares the tax returns of the president of a corporation the CPA audits, without charging the president.

mini standard deduction

The standard deduction for an individual for whom an exemption can be claimed on another person's tax return is generally limited to the greater of: $1,050, or. The individual's earned income for the year plus $350 (but not more than the regular standard deduction amount, generally $6,300).

you own 80% interest in a partnership, partnership sold you a land and recorded 6000 loss, you must

add the 80% of 6000 loss to your distributive share of ordinary income

a corporation's retained earning

is increased by net income per book decreased by dividends paid and debit adjustment to the beginning balance of RE for correction of an accounting error.

these liens generally require the lien-holders to give notice of legal action before selling the debtor's property to satisfy the debt

mechanic Lien an artisan's lien

a covenant not to compete is taxed as

ordinary income

an S corporation's tax items are allocated to shareholders on an

per share' and per day basis

the market value to be used in computing the buyer's damage would be

the market value at the time the buyer learned of the breach

stock held for less than 12 months and donated to charity is valued at

the original purchase price rather than FMV, the stock is not capital gain property because it was not held for more than a year. if long term, then its valued at FMV

a partnership is a legal entity that can own property in its own name

the partners can also have joint and several liability for all debts whether they are based in contract or tort

under the bankruptcy code, one of the elements that must be established in order for the trustee in bankruptcy to void a preferential transfer to a creditor who is not an insider is that

the transferee-creditor received more than he would have received in a liquidation proceeding under the bankruptcy code. another element is that transfer involved an antecedent debt, no preferential transfer exists if there is a contemporaneous exchange for new value

a buyer in the ordinary course of business will not be subjected to the creditor's security interest

this rule applies to purchasing for commercial and consumer uses.

Amounts received by shareholders in complete liquidation of a corporation are treated as received in exchange for stock,

generally resulting in capital gain or loss because the stock was held as an investment

when a corporation has unused net capital loss that is carried back or carried forward to another tax year

it is treated as a short-term capital loss whether or not it was short-term when sustained.

To satisfy the consideration requirement for a valid contract, the consideration exchanged by the parties must be

legally sufficient

single-member limited liability company liability

limited liability

notice of revocation must be received by the offeree before the revocation becomes effective

on the other hand, notice of acceptance must be received by the offeror before the acceptance becomes effective.

the adoption credit for adoptions finalized in 2018 is up to

$13810 per child, This is a non-refundable tax credit, which means that you must have federal tax liability in order to claim the credit. You have five years to use the full amount of credit to reduce your federal tax liability.

deductible interest expense include

interest on the mortgage loan and interest on the line of credit loan the interest expense on the loan obtained to purchase municipal bonds is not deductible because municipal bonds produce tax-exempt income

Jeff is single and he provides 100% support for his mother. Jeff lives in Tennessee and his mom, Betty, lives in Illinois.

Betty is a dependent and a qualifying relative since Jeff provides over 50% of her support

Terrence and Lavonne are married. During 2018, Dustin, Terrence's nephew, lived with them for the entire year. Dustin has income from a part-time job of $6,000. He is 22 years old and is a full time student. Dustin provides 35% of his total support.

Dustin is a qualifying child because QC includes siblings, nieces and nephews and grandchildren. Dustin meets the age test for a QC since he is a full time student and is less than 24. He also does not provides more than 50% of his own support.

Self-employment taxes paid.

Fifty percent deductible on Form 1040 to arrive at adjusted gross income.

Qualifying contributions to a simplified employee pension plan.

Fully deductible on Form 1040 to arrive at adjusted gross income.

black is a purchaser in the ordinary course of business and will defeat bank's prior perfected security interest in cutters inventory

the fact that black is buying for business purposes would not affect its status as purchaser in the ordinary course of business

gambling losses is deductible to the extent of

the gambling winnings

Net operating loss in 2017 must be carried back 2 years and carried forward 20 years

NOL carry back is eliminated now and NOL deduction is limited to 80% of taxable income

This year Jerry sent his mother, Nell, $3,000 to help with her support. Nell's only other source of support was from a rental property she owns that had revenue of $4,900 and net income of $2,500.

Nell is not a dependent because her gross income is over the exemption amount of $4150

Ming created a joint bank account for herself and her son, Zhi. There is a gift to Zhi when

Zhi draws on account for his own benefit

a simple trust is one that 1) required to distribute income every year. 2) has no beneficiary that are qualifying charitable organizations, 3) makes no distributions of trust corpus

a complex trust is any trust that is not a simple trust.

in computing the net operating loss for the current year, the Dividends received deduction would be fully allowed

but the NOL deduction (carryover from the prior year) would not be allowed

under the UNICAP rules, all costs incurred in purchasing or holding inventory for resale must be

capitalized as part of the cost of inventory. these costs include the costs of purchasing, handling, processing, repackaging, assembly and warehousing service costs such as marketing, selling, distribution, and office maintenance are immediately deductible and need not be capitalized as part of the inventory.

book income to taxable income element

deduct interest income from municipal bonds add back net capital loss add back federal income tax expenses

Taxes payable under the Federal Unemployment Tax Act (FUTA) are A. Calculated as a fixed percentage of all compensation paid to an employee. B. Deductible by the employer as a business expense for federal income tax purposes. C. Payable by employers for all employees. D. Withheld from the wages of all covered employees.

deductible by the employer as a business expense for federal income tax purposes. not deductible for the employee because the employee does not pay them the taxable payable under the FUTA are calculated as a fixed % of only the first $7000 of wages of each employee mcq: emen-0002

credit card charge in current year and payment not yet made to the credit card company is

deductible for the current year's income tax return

dividends can be taxed at ordinary income tax rates or at preferred long term capital gains tax rates

dividends that qualify for the lower long-term capital rates are called qualified dividends. must be held the stock for more than 60 days during the 121-day period that begins 60 days before the ex-dividend date to be considered a qualified dividend.

syndication expenses may neither be amortized nor deducted

eg of syndication expenses include legal advice concerning issuing stock certificates.

the corporation may elect to amortize organizational expense over the 180 month period beginning with the month in which the corporation begins operations, however, the first $5000 of organizational expenses may be deducted immediately,

eg. of organization cost includes legal advice concerning the formation of corporation and transferring of assets.

constructive dividend

eg. selling land with a FMV of $80k to a shareholder for $50k, the $30k is considered dividend income to the shareholder

for services donated to the partnership you must recognized gain

equal to the value of service contributed. the value of service is added to your basis

business related expenses are deductible

if you are self-employed, but not when you're working for a business.

an incorporated exempt organization subject to tax on its unrelated business income

must comply with the code provisions regarding installment payments of estimated income tax by corporations. this means that an exempt organization must make quarterly tax payments if it expects its estimated tax on its unrelated business income to be $500 or more.

a CPA will not be held liable if

negligence cannot be proved

a CPA may not provide working papers without consent or a subpoena to

neither IRS or FASB

this is an example of deductible medical expenses for tax purposes

premiums paid for medicare b supplemental medical insurance

under the chapter 11 bankruptcy, this action is necessary before the court may confirm a reorganization plan

provision for full payment of admin expenses and although a trustee may be appointed, it is often not appointed.

proxy statements are sent with the

proxy solicitations

both municipal bonds and nfp securites are exempted from

registration under the 1933act

income before special deduction includes

sales-cost of sales+dividend received

Above the line deductions for AGI

schedule C or F business deductions rental deductions , e,g real estate taxes stock losses student loans interests paid alimony contribution to self-employed retirement plan self-employment taxes

this dividend is not defined as a distribution

stock dividends.

ratification of contract may be accomplished by

words and and actions

the law of joint ventures is similar to that of

general partnerships.

the 2016 standard deduction for a trust or an estate in the fiduciary income tax return is

$0 on the other hand, a personal exemption is allowed for an estate or trust on the fiduciary income tax return. the personal exemption is $600 for an estate, $300 for a trust required to distribute all income currently, and $100 for all other trusts

During 2018, Blake transferred a corporate bond with a face amount and fair market value of $20,000 to a trust for the benefit of her 16 year old child. Annual interest on this bond is $2,000, which is to be accumulated in the trust and distributed to the child on reaching the age of 21. The bond is then to be distributed to the donor or her successor-in-interest in liquidation of the trust. Present value of the total interest to be received by the child is $8,710. The amount of the gift that is excludable from taxable gifts is

$0, all taxable in the future Since the interest income resulting from the bond transferred to the trust will be accumulated and distributed to the child in the future upon reaching the age of twenty-one, the gift (represented by the $8,710 present value of the interest to be received by the child at age twenty-one) is a gift of a future interest and is not eligible to be offset by an annual exclusion.

Earl Cook, who worked as a machinist for Precision Corp., loaned Precision $1,000 in 2015. Cook did not own any of Precision's stock, and the loan was not a condition of Cook's employment by Precision. In 2018, Precision declared bankruptcy, and Cook's note receivable from Precision became worthless. What loss can Cook claim on his 2018 income tax return?

$1,000 short-term capital loss. Cook's $1,000 loss will be treated as a nonbusiness bad debt, deductible as a short-term capital loss. The loss is not a business bad debt because Cook was not in the business of lending money, nor was the loan required as a condition of Cook's employment. Since Cook owned no stock in Precision, the loss could not be deemed to be a loss from worthless stock, deductible as a long-term capital loss.

Water Works had a long-standing policy of offering employees $100 for suggestions actually used. Due to inflation and a decline in the level and quality of suggestions received, Water Works decided to increase the award to $500. Several suggestions were under consideration at that time. Two days prior to the public announcement of the increase to $500, a suggestion by Farber was accepted and put into use. Farber is seeking to collect $500. Farber is entitled to

$100 in accordance with the original offer. An offeree cannot accept an offer unless the offeree knows of the existence of the offer; otherwise there is no objective meeting of the minds. Farber does not know of the increase of the monetary award to $500, and submitted his suggestion as an acceptance to the terms of the offer of which he knew (i.e., the $100). Even though Water Works later raised the amount and will use the suggestion in the later period, Farber only has a right to receive $100.

a tax exempt organization that has receipts below a specified amount does not have to file an annual informational return, what is that amount?

$25000

a self-employed individual must file an income tax return if net earnings from self employment are ____ or more

$400

(child and dependent care credit) the amount of qualifying expense is limited to $3000 for one qualifying individual and

$6000 for more than one qualifying individual, note that qualifying expenses are also limited to actual amount paid and the wages earned of the lowest paid spouse the qualifying expense of $6000 are multiplied by the applicable percentage, which ranges from 20-35%. the 35 maximum is reduced by one percentage point for each $200 that AGI exceeds 15000. once AGI exceeds 43000 the percentage is reduce to its minimum of 20%

Generally, to avoid a penalty for the underpayment of estimated taxes, a corporation's quarterly estimated payments must be at least equal to the least of

(1) 100% of the tax shown on the current year's tax return, (2) 100% of the tax that would be due by placing income for specified monthly periods on an annualized basis, or (3) 100% of the tax shown on the corporation's return for the preceding year, provided the preceding year showed a positive tax liability and consisted of 12 months.

A corporate stock redemption is treated as an exchange, generally resulting in capital gain or loss treatment to a shareholder if the redemption meets any one of five tests. Redemptions qualifying for exchange treatment include

(1) a redemption that is not essentially equivalent to a dividend, (2) a redemption that is substantially disproportionate, (3) a redemption that completely terminates a shareholder's interest, (4) a redemption of a noncorporate shareholder in a partial liquidation, and (5) a redemption to pay death taxes. If none of the above five tests are met, the redemption proceeds are generally treated as a dividend.

a member of the AICPA may be subject to expulsion or suspension from membership without hearing except 1The member's license to practice is revoked by a state board as a disciplinary measure. 2The member files a fraudulent tax return. 3The member is convicted of a crime punishable by imprisonment for 5 years. 4.The member is prohibited from doing any work on audits of issuers by the PCAOB for 3 years.

.The member is prohibited from doing any work on audits of issuers by the PCAOB for 3 years. This is not a situation that is indicated as a likely reason for expulsion or suspension without a hearing.

in a like-kind exchange, gain is recognized to the extent of the lesser of

1) boot received 2) gain realized

qualifying widower filing status is available for the two years following the year of a spouse's death if

1) the surviving spouse was eligible to file a joint return in the year of the spouse's death 2. does not remarry before the end of the current year 3) the surviving spouse pay over 50% of the cost of maintaining a household that is the principal home for the entire year of the surviving spouse's dependent child

which forms of business organization can have characteristics of both corporation and general partnership

1. LLC (yes) 2. s corp (yes) LLC provides for limited liability similar to a corporation can has limited duration of existence, like that of a partnership. the LLC can be taxed similar to a partnership if formed to do so. the s corp has the limited liability of the corporation but is taxed similar to a partnership

under the Revised uniform partnership act, in these cases will property be deemed to be partnership property

1. a partners acquires property in the partnership name 2. a partner acquires title to it in his own name using partnership funds. note that a partner may use property in the partnership business without it becoming partnership partnership

what events will follow the filing of a chapter 7 involuntary petition

1. a trustee will be appointed: yes 2 a stay against creditor collection proceedings will go into effect: yes

medical expenses for a decedent paid after death

1. are deductible on decedent's final return if paid during the one year period after death and the estate elects to treat them as paid by dededent 2. are not deductible.

which of the following limitations may apply in determining the allowable deduction for a partner's distributive share of partnership losses

1. at risk (yes) 2. passive loss (yes)

a disclaimer cannot be used to escape liability for

1. breach of contract 2. negligence 4. fraud.

these expense items should be reported on schedule M1 of the corporate income tax (form 1120) showing the reconciliation of income per books with income per return

1. deduction for a net capital loss (yes) 2. business meals for executive out of town travel (yes, 50% deduction) Since only 50% of business meals is deductible for tax purposes, 50% of business meals would be added back to book income to arrive at taxable income on Schedule M-1.

elements necessary for a promissory estoppel are

1. detrimental reliance on a promise 2. reliance on the promise is reasonable and foreseeable 3. damage results if the promise is not enforced.

items deductible for tax purposes but not for book purporses

1. fed income tax 2. political contribution 3. business entertainment 4. premiums paid to key officers 5. interest income from bonds sold by city

bankruptcy priority

1. fee for appraising assets 2. wages to employees 3. state and fed taxes 4. general, unsecured claims

Personal holding company (PHC)

1. five or fewer individual owning more than 50% of the stock during the last half of its taxable year 2. at least 60% of its AGI is derived from investment sources (dividends, interest, rents)

in general, the 3rd party beneficiary rule as applied to a CPA's legal liability in conducting an audit is relevant to which of the following causes of action against a CPA

1. fraud: no 2. constructive fraud: no 3. negligence: yes Since a CPA is generally liable to all third parties, including foreseen and foreseeable third parties, for fraud and constructive fraud, the third-party (primary) beneficiary rule is relevant only in those cases based on negligence.

in the event of a default, the creditor can go after

1. full amount immediately from the sureties when the corporation defaults 2. demand full payment immediately from the sureties even if reuter does not try to receove any amount from the collateral 3. try to sell the collateral and the remainder from the sureties,

tax preparers who aid and abet federal tax evasion are subject to

1. injunction to be prohibited from acting as tax preparers : yes 2. general federal criminal prosecution: yes

items that are reported on schedule k includes

1. interest income 2. municipal interest income 3. section 1231 gain 4. charitable contribution 5. life insurance premium on partners' lives

interests reported as interest income on the tax return include

1. interest on refund of federal income tax for 2016 2. interest on award for personal injuries sustained in an automobile accident during 2015 3. interest on US savings bonds (series HH)

DAC foundation awarded Kent 75000 in recognition of lifelong literary achievement. Kent was not required to render future services as a condition to receive the $75000, these conditions must have been met for the award to be excluded from kent's gross income

1. kent was selected for the award by DAC without any action on Kent's part 2. pursuant to kent's designation, DAC paid the amount of the award either to a government unit or to a charitable organization.

MACRS conventions

1. mid-year convention: applies to all property except real estate 2. mid month convention: applies to real estate only 3. mid quarter convention: applies to some personal property (when major acquisitions are made in the final quarter)

a partner's interest in specific partnership property is

1. not assignable to the partner's individual creditors 2. subject to attachment by the partner's individual creditors.

who has the fiduciary duty in a corporation

1. officer and directors of a corporation 2. majority shareholders to the minority shareholders because majority shareholders can exercise a lot of power

eg of deductible taxes for itemized deductions

1. property tax on residence 2. state personal property taxes assessed on the value of automobile 3. property taxes on land held for long term appreciation.

preferences items for AMT tax include

1. tax exempt interest from private activity bonds 2. addition of certain income from incentive stock options 3. the excess of accelerated depreciation claimed when compared to a hypothetical straight line amount on property placed in service before 1987

which of the following circumstances may permit the piercing the corporate veil of a closely held corporation and thus may cause its shareholders to be held personally liable

1. the corporation is thinly capitalized. the decision to grant a security interest is a matter of negotiation between the shareholder and the corporation, it is a private contract matter between the two parties, whether a security interest is granted or not, provides no basis for piercing the corporate veil

these statements are correct concerning issuers of securities registered under the 1934 act

1. the issuers must have each form 10-k audited by an CPA 2. form 10q just needed to be reviewed, not audited.

these statements are true of a professional corporations under the various state laws

1. the professionals in the corporation have personal liability for their professional acts 2. normally under state laws, only licensed professionals are permitted to own shares in professional corporations.

under the sales article of UCC, the remedies available to a seller when a buyer breaches a contract for the sale of goods may include

1. the right to resell goods identified to the contract (yes) 2. the right to stop a carrier from delivering the goods (yes)

attachment of a security interest in personal property occurs when

1. the secured party gives value 2. the debtor has rights in the collateral 3. a record of the security agreement must exist.

the transaction of the director wishes to have with the corporation is not a conflict of interest in any of these are true

1. trx is fair and reasonable 2. shareholders are given the relevant facts and they approved it by a majority vote 3. the BOD are given the relevant facts and they approve it by a majority vote of the disinterested members of the BOD.

Upon a corporate formation, no gain or loss is recognized to the shareholders or the corporation if the following three requirements are met:

1.The shareholders contributing property to the corporation own, as a group, at least 80% of the voting stock (and at least 80% of the number of shares for each class of nonvoting stock), 2.The shareholder contributes property (i.e., not services) to the corporation, and 3.The shareholder receives only stock of the corporation in exchange. 4. if cash was received, you recognized gain equal to the amount of cash

in a multiple support agreement, each person must contribute more than ___% of the support

10%

a tax exempt organization does not have to pay income taxes on unrelated business income if it does not exceed what amount

1000

a kid's basic standard deduction is limited to the greater of

1050, or the earned income of $1700, plus $350

under regulation D of the 1933 act, the maximum time period during which an exempt offering may be made

12 months

to qualify for the child and dependent care credit, the expenses must be incurred to care for a dependent under the age of

13

sec 291 requires the amount of ordinary income on the disposition of sec 1250 property by corporations be increased by

20% of the additional amount that would have been ordinary income if the property had insated been sec 1245 property. if the building had been sec 1245 property the amount of recapture would have been 30000. thus the sec 291 ordinary income is 30000*0.2=6000. the remaining 44000 is sec. 1231 gain.

a corporation tax rate right now is

21%

depreciation recapture rate is

25%

federal limit for garnishment of wages

25% of the debtor's wages

business interest expense is limited to

30% of the business adjusted taxable income, however, you are not subject to the limitation if your gross receipts do not exceed the $25mil average receipts test

a revocation of an S selection will be effective if it is signed by shareholders owning more than ___% of the S corporation's outstanding stocks

50

a qualifying organization normally having annual gross receipts of not more than a specified amount does not have to file to get a tax-exempt status, what is that amount?

5000

the filing fees incident to the creation of the partnership are organizational expenditures, a partnership may deduct up to

5000 of organizational expenditures for the tax year in which the partnership begins business, any remaining expenditures are deducted ratably over the 180 month period beginning with the month in which the partnership begins business. less than $5000, fully deductible.

the income test is met if the personal holding company income is consisted of ___% of adjusted ordinary gross income

60%

cash gift to church and donation to community colleges are only subject to the

60% of AGI limitation

upon default, the secured party normally has the right to retain(or sell the collateral to satisfy the obligation. however, the secured party cannot retain the collateral if it is consumer goods and the debtor has paid

60% or more of the obligation, in such a case, the debtor is entitle to a compulsory disposition of the goods.

a self-employed taxpayer is allowed to deemed deduction equal to

7.65% of self-employment earnings in computing the amount of net earnings upon which tax is based.

security interests given by debtors to secure antecedent debts may be set aside by the trustee in bankruptcy when given within the previous

90 days.

Section 351 Tax-Free Exchange

A provision that allows unincorporated entities to incorporate without significant tax ramifications by making exchanges of property or cash for equity nontaxable, provided only stock is received in exchange for the property and the party transferring the property and cash has control, consisting of at least 80% ownership when the exchange is complete. Services are excluded from the definition of property and are considered taxable exchanges.

Possession of Collateral by Secured Party

A secured party may perfect a security interest in most types of collateral simply by taking posession of the collateral. The security interest is perfected from the moment of possession without relation back to the time of attachment. Perfection continues only so long as possession is retained. gives you the best position in the events of default

Nolan designed Timber Partnership's new building. Nolan received an interest in the partnership for the services. Nolan's normal billing for these services would be $80,000 and the fair market value of the partnership interest Nolan received is $120,000. What amount of income should Nolan report?

A taxpayer must recognize income when a capital interest in a partnership is received as compensation for services rendered. the amount recognized is equal to the fmv of partnership interest received, in this case $120000

Accumulated Adjustments Account (AAA)

AAA represents the cumulative total of an S corporation's income items, less expenses and distributions, the amount of increased by interests and dividends and reduced by charitable contributions and distributions, capital contributions would have not effect on the AAA

If an individual never made a nondeductible contribution to a traditional IRA, then any distributions from the IRA are fully taxable as ordinary income. Also, if the individual is under age 59½, the distribution is generally subject to the 10% penalty tax for early distributions. Here, the $30,000 distribution would be taxed at the taxpayer's marginal rate of 35% resulting in a tax of $10,500.

Additionally, there will be a penalty tax of 10% × $30,000 = $3,000, because of having received the distribution before age 59½, resulting in a total tax liability of $13,500. withdraw early, pay 10% penalty

Section 197 intangibles

Amortized over 15 years

classification of a principal as disclosed, undisclosed, or partially disclosed affects the contractual liability of the agent toward third parties.

An agent representing an undisclosed principal has the same implied authority as an agent representing a disclosed principal.

A calendar year C corporation discovers an error resulting in overpayment of tax on the return for year 1 filed on 2/17/year 2. When does the statute of limitations for filing an amended return expire?

April 15, year 5, in general, you have three years to file the amended tax return.

Pell is the principal and Astor is the agent in an agency coupled with an interest. In the absence of a contractual provision relating to the duration of the agency, who has the right to terminate the agency before the interest has expired?

Astor, the agent coupled with interest, the authority of the agent (Astor) is irrevocable by the principal (Pell). If there is no time period specified for the agency, then the agent (Astor) may terminate at any time without liability, regardless of the type of agency.

forming an agency relationship requires that

Both the principal and agent consent to the agency

this entity may adopt any tax year-end

C corporation, the requirement is to determine which entity is free to adopt any tax year it chooses, a C corporation is free to adopt calendar or any fiscal year that it chooses, on the otherhand, restrictions apply to the adoption of a taxable year by s corporations, llcs and trusts in order to prevent the deferral of income to owners and beneficiaries that could otherwise be achieved, an S corporation's income is generally passed through to shareholders at the end of the S corporation year, as a result, an S corporation generally must adopt a calendar year, and can request permission to adopt a fiscal year only if the corporation established a business purpose. An LLC that does not elect to be taxed as an association is a partnership for tax purposes and is subject to the same restrictions regarding the adoption of a taxable year that apply to other partnership. Generally, an LLC must adopt the same taxable year as used by its one or more owners who have an aggregate interest in LLC profits and capital exceeding 50%, distributions from trusts are taxed to beneficiaries in the beneficiary's tax year in which the trust's year ends, As a result, trusts are generally required to adopt a calendar year.

In an action for negligence against a CPA, "the custom of the profession" standard is used at least to some extent in determining whether the CPA is negligent. Which of the following statements describes how this standard is applied?

Despite a CPA's adherence to the custom of the profession, negligence may nevertheless be present. In certain cases the SEC and the courts have held that even though the CPA adhered to GAAP and GAAS, negligence was present when misleading financial statements resulted. The SEC and the courts are showing an increasing proclivity to not allow customs of the profession to preclude using a case of negligence against CPAs.

At a confidential meeting, an audit client informed a CPA about the client's illegal insider-trading actions. A year later, the CPA was subpoenaed to appear in federal court to testify in a criminal trial against the client. The CPA was asked to testify to the meeting between the CPA and the client. After receiving immunity, the CPA should do which of the following?

Discuss the entire conversation including the illegal acts. the code of professional conducts tells a CPA to comply with an enforceable subpoena

a deduction for a net operating loss carryover is not allowed in computing a NOL

Furthermore, a deduction for charitable contributions is generally not allowed, since the charitable contributions deduction is limited to 10% of taxable income before the charitable contributions and dividends-received deductions.

Adjusted Ordinary Gross Income (AOGI):

Gross income excluding capital and 1231 gains and reduced by expenses associated with the production of rent and royalty income

If both cash and noncash property are received in a single distribution, the cash reduces the basis of the partner's interest before the distribution of noncash property.

Here, no gain is recognized because the cash distribution ($11,000) does not exceed the basis of Reed's partnership interest before the distribution ($25,000).

if an agent acts without authority, neither the principal nor the third party is bound to perform the contract.

However, if principal ratified agent's unauthorized act before 3 party withdrew from the contract, 3rd party would be bound by the agreement.

Schedule M-1 generally provides a reconciliation of a corporation's income per books with the corporation's taxable income before the NOL and dividends received deductions. Since a net capital loss deducted per books would not be deductible for tax purposes, the net capital loss would be added back to book income on Schedule M-1.

However, since out-of-town lodging expenses are deductible for both book and tax purposes, the expenses would not appear on Schedule M-1.

When a corporation elects to be a Subchapter S corporation, which of the following statements is (are) true regarding the federal tax treatment of the corporation's income or loss?

III. The shareholders report the corporation's income on their tax returns even if the income is not distributed to them. IV. The shareholders generally report the corporation's loss on their tax returns. When a corporation elects to be a Subchapter S corporation, the corporate income and loss flow through to the income tax returns of the individual shareholders even when the income is not distributed to them. s crop is not taxed a corporate level

Johns owns 400 shares of Abco Corp. cumulative preferred stock. In the absence of any specific contrary provisions in Abco's Articles of Incorporation,

If Abco declares a cash dividend on its preferred stock, Johns becomes an unsecured creditor of Abco.

Income in Respect of a Decedent (IRD)

Income received by a beneficiary that was due the decedent but wasn't paid to the decedent before death. included in the dedecent's gross estate if collected by the executor of the estate: must be included in the estate's fiduciary income tax return

What is the standard that must be established to prove a violation of the antifraud provisions of Rule 10b-5 of the Securities Exchange Act of 1934?

Intentional misconduct.it must be proven that a false statement or omission of material fact was made, and the individual knew it was false or acted with reckless disregard for the truth. This is equivalent to intentional misconduct.

Partnership incurred interest expense on partnership investment indebtedness

Interest expense from investment indebtedness is a separately stated item and potentially deductible to the partner(s) as an itemized deduction (investment paid on schedule A of form 1040).

Gilbert borrowed $10,000 from Merchant National Bank and signed a negotiable promissory note which contained an acceleration clause. In addition, securities valued at $11,000 at the time of the loan were pledged as collateral. Gilbert has defaulted on the loan repayments. At the time of default, $9,250, plus interest of $450, was due, and the securities had a value of $8,000. Merchant

Is entitled to proceed against Gilbert on either the note or the collateral or both. collateral provides another source of payment beside the debtor's promise. the bank does not need to proceed against first against the collateral before proceeding against Gilbert personally.

Yancie took her bike in to Pete's Bike Sales and Repair to have it repaired. Pete said he would need to have her leave it for two days. The next day, one of Pete's employees sold Yancie's bike to Jake. Jake paid for the bike with a credit card, unaware that Pete did not own the bike. Which of the following is correct?

Jake has title to the bike. yancie can recover the value of the bike from Pete

cannot use the preceding tax year method if you have a

NET operating loss for the preceding year, need to show a positive tax liability and consist of 12 months

Partnership made a cash distribution to partnership C

No tax effect but it reduces the partners' basis.

Partnership paid fines and penalties

Not tax deductible and it reduces their basis as an expense.

A sole proprietor of a farm implement store sold a truck for $15,000 that had been used to make service calls. The truck cost $30,000 three years ago, and $21,360 depreciation was taken. What is the appropriate classification of the $6,360 gain for tax purposes?

Ordinary gain. A business truck is Sec. 1245 property and a gain on its disposition is subject to recapture as ordinary income to the extent of all depreciation previously deducted. Since $21,360 of depreciation had been deducted, all $6,360 of gain is classified as ordinary income.

the charitable contribution deduction for art subject purchased is limited to the

amount excess of its fair market value.

A nonliquidating distribution of unappreciated inventory reduces the recipient partner's basis in his partnership interest.

Partnership distributions are generally nontaxable and reduce the recipient partner's basis by the adjusted basis of the property distributed.

Section 1245 property

Personal property that can be depreciated. This type of property includes tangible personal property, such as furniture and equipment, that is subject to depreciation, or intangible personal property, such as a patent or license, that is subject to amortization.

Which of the major transaction exemptions enables an issuing firm to test the waters before actually holding an offering?

Regulation A: REG A allow an issuer to test the waters and cancel the offer if there appears to be insufficient interest by investors.

deductible for state income taxes on Sara's 2018 federal income tax return?

amount withheld by employer plus the total of estimated payments made

items used to compute partnership ordinary income loss

Sales income $100,000 Cost of goods sold (70,000) Advertising expense (4,000) Supplies expense (4,000) Depreciation expense (4,000) Taxes (9,000) Guaranteed payment (12,000) Ordinary loss (3,000)

Self-employment income includes an individual's net earnings from a trade or business carried on as sole proprietor or as an independent contractor. The term also includes a partner's distributive share of partnership ordinary income or loss from trade or business activities, as well as guaranteed payments received by a partner for services rendered to a partnership.

Self-employment income excludes gains and losses from the disposition of property used in a trade or business, as well as a shareholder's share of ordinary income from an S corporation.

partnership may elect to have a tax year other than

generally required tax year if the deferral period for the tax year elected does not exceed 3 months.

a corporation's contribution base is its taxable income before the charitable deduction, the dividends received deduction, and before deductions for capital loss carrybacks.

Since Gero had operating income of $160,000 after deducting $10,000 of contributions, its contribution base would be $160,000 + $10,000 + $2,000 dividends = $172,000.

In 2014, Ross was granted an incentive stock option (ISO) by her employer as part of an executive compensation package. Ross exercised the ISO in 2017 and sold the stock in 2019 at a gain. Ross was subject to regular tax for the year in which the

Stock was sold.

An accountant compiled the unaudited financial statements for Taylor Company, a nonissuer company. The financial statements contained a material misstatement that was not discovered in the compilation. The accountant issued a report that stated that the financial statements were fairly stated based on the limited evidence that he collected. Which of the following is true about the accountant's liability to a third party who relies on the financial statements?

The accountant will likely be held liable because an appropriately worded report was not issued. The report overstated the accountant's level of work.

Melvin Crane is 66 years old, and his wife, Matilda, is 65. They filed a joint income tax return for 2018, reporting an adjusted gross income of $22,200, on which they owed a tax of $61. They received $3,000 from Social Security benefits in 2018. How much can they claim on Form 1040 in 2018, as a credit for the elderly?

The amount of credit (limited to tax liability) is 15% of an initial amount reduced by Social Security and 50% of AGI in excess of $10,000. Here, the credit is the lesser of (1) the taxpayers' tax liability of $61, or (2) 15% [$7,500 − $3,000 − (.50)($22,200 − $10,000)] = $0.

Lund Pizza Company gave specific rules to employees including obeying all traffic laws while delivering pizzas. An employee, while delivering pizzas one day, negligently went through a red light and injured the driver of another car. Which of the following is correct?

The injured driver can recover from either Lund or the employee or both. Even though the employee was breaking one of Lund's rules, they can both be held liable, either individually or together.

an S corporation cannot own stock of another S corporation

an S corp is allowed to be a partner in another partnership, and my own any percentage of stock of a C corp, as well owned 100% of the stock of a qualified subchapter s subsidiary.

The earned income of a child of any age and the unearned income of a child 18 years or older as of the end of the tax year is taxed at the child's own tax rates. However, the unearned income of a child under age 18 in excess of a threshold amount is generally taxed at the trust and estate rates.

The threshold amount is subject to change because it is indexed for inflation, but it is normally twice the amount of the applicable standard deduction for a dependent who has only unearned income. Since the multiple-choice item assumes the applicable standard deduction for the child is $1,000, the applicable threshold would be $1,000 × 2 = $2,000. As a result, $3,000 interest income − $2,000 threshold = $1,000 of the child's interest income would be taxed using the trust and estate rates.

An S corporation engaged in manufacturing has a year-end of June 30, 2018. Revenue consistently has been more than $30 million under both cash and accrual basis of accounting. The stockholders would like to change the tax status of the corporation to a C corporation using the cash basis with the same year-end. Which of the following statements is correct if it changes to a C corporation?

The year-end will be June 30, using the accrual basis of accounting. because a C corporation may elect to use either a calendar year or a fiscal year as its annual accounting period. no cash method for C corp if annual receipt is over 25million

Upon declaration, a cash dividend on preferred stock becomes a legal debt of the corporation, and the preferred shareholders become unsecured creditors of the corporation.

Therefore, Sky will be liable to West as an unsecured creditor for $35,000 which is the amount of the declared dividends.

Since there are fewer than 12 creditors, it is true that only one creditor is needed to file the petition. However, no one creditor is owed at least $15,325 of unsecured debt.

Therefore, the claims can be aggregated to total at least $15,325 of unsecured debt. The only way this can be accomplished is by aggregating the claims of all four creditors.

Under the Revised Uniform Partnership Act, the partners have joint and several liability for breaches of contract as well as for torts.

This allows third parties to sue them together or separately.

if you intend to solicit proxies relating to annual meeting at which directors will be elected, you are subject to the registration and reporting requirements of the 1934 act, as a result you must furnish your shareholders with

an annual report containing audited balance sheets for the 2 most recent years.

What is the appropriate tax treatment if the Partnership sold an investment held for more than one year at a gain?

Treated as separately stated income by the partnership, taxable to the partner. Partners must pay tax on the net income of the partnership, even if the partnership retains the income. Any amount that a partner does not withdraw increases that partner's basis; distributions decrease it. Therefore, distributions to partners per se are not taxed unless the distribution exceeds the partners' adjusted basis in the partnership, in which case, the excess is taxable; otherwise the draw is considered a return of capital.

Beginning in 2018, net unearned income in excess of $2,100 is taxed at the

Trust and estate return rates due to the Kiddie tax rules for children 18 and under.

this entity is entitled to the net operating loss deduction

Trusts and estates trusts and estates may have an NOL that can be carried back and carried forward and used as a deduction. Furthermore, for the year in which a trust or estate terminates, any remaining unused NOL passes through to the beneficiaries who succeed to the assets of the trust or estate.

section 351

Under section 351(a) no gain or loss is recognized (reported) provided: You receive ONLY STOCK in exchange for your property, and You are in CONTROL of the corporation immediately after the exchange.

The limited liability of the shareholder of a closely held corporation will most likely be disregarded if the shareholders

Undercapitalized the corporation when it was formed. the court will pierce the corporate veil if the corporation is used to defraud people. such as established a corporation with less capital than it needs to pay debts

Walker tells Side that she will hire him for 10 months' work for $6,000 per month starting 6 months from now. Side accepts. When Side proceeds to work for Walker, Walker refuses, pointing out that their contract was not in writing. Side admits the lack of a written contract but claims they still have an enforceable contract. Who wins?

Walker because this contract needed to be in writing. This contract could not be completed until 16 months after the making of the contract. Therefore, it had to be written since it fell under the Statute of Frauds since it could not be completed within 1 year of the making of the contract.

a good faith purchaser for value at a private sale will take the property free from?

any security interest or subordinate liens in the property, but remains subject to security interests which are senior to that being discharged at the sale.

the family tax credit applies to

a 30 year old nephew who is a qualifying relative, covers older relatives, child tax credit only covers child dependents up through the age of 16.

according to the rules of bankruptcy procedure, it is necessary to file a proof of claim against the debtor' estate, the filing must be timely(within a 6 month-period) or the claim will be barred

a claim that is file on time is give prima facie (at first appearance) validity and is approved unless there is an objection by one of the creditors, the filing would include a statement of compensation paid or agreed.

acceleration clause

a contract provision that allows a lender to require a borrower to repay all of an outstanding loan if certain requirements are not met.

under a Chapter 11 reorganization

a debtor is allowed to remain in possession of its business unless the court upon request by a party in interest appoints a trustee to take over management of the debtor's business. court would allowed when the gross mismanagement of the debtor's business has occurred, or that the takeover by a trustee would be in the best interest of the debtor's estate

if the value of the collateral is not enough to satisfy a claim, the secured party becomes

a general creditor for the remaining balances. thus the creditor does have a claim for the balance

you can use the doctrine of substantial performance if you made

a good faith effort, not an intentional breach of contract.

in order to be treated as alimony,

a payment must be made in cash and be received by or on behalf of the payee spouse, furthermore, cash payments must be required to terminate upon the death of the payee spouse to be treated as alimony. mortgage payments are not treated as alimony because they will be made throughout the duration of the mortgage period and will not terminate in your ex-spouse's death

in a contract entered fraudulently

a person with a voidable title may transfer good title to a good-faith purchaser, thus able has no course of action against Gold, a good-faith purchaser, however, Able is entitle is recover damages from Baker.

a promise to donate money to a charity on which the charity relied in incurring large expenditures will be legally binding despite lack of consideration

a promise to donate money to a charity which the charity relied upon in incurring large expenditures is a situation involving promissory estoppel. promissory estoppel acts as a substitute for consideration and renders the promise enforceable,

co sureties exist when there is more than one surety guaranteeing the same obligation of the principal debtor. unless the creditor specifically reserves his rights,

a release of a cosurety by the creditor will release the other cosurety to the extent of the released cosurety's pro rata share of debt liability.

although a rental activity is defined as a passive activity,

a special rule allows the individual to offset up to $25000 of income that is not from passive activities by losses from a rental real estate activity if the individual *actively participates in the rental real estate activity , However, this special $25,000 allowance is reduced by 50% of the taxpayer's AGI in excess of $100,000 and is fully phased out when AGI exceeds $150,000

proxy statement

a statement required of a firm when soliciting shareholder votes.

the basis for an S corporation stock is increased by the flow through of all income (including nontaxable income),

and is reduced by distributions as well as the flow-through of all deductions( including nondeductible items) e.g increased by net business income and nontaxble municipal bond interest income

Article of Incorporation

a written legal document that defines ownership and operating procedures and conditions for the business 1. number of shares authorized 2. name and address of each incorporator 3.the name and address of the registered agent of the corporation

a corporation must make estimated tax payments unless it is tax liability can reasonably be expected to be less than $500

a. corporation's estimated tax is its expected tax liability (including the accumulated earnings tax) less its allowable tax credits.

ultra vires acts are

acts that are beyond the scope of corporate powers

For a purchaser of land to avoid a contract with the seller based on duress, it must be shown that the seller's improper threats

actually induced the purchaser to assent to the contract

Since Burg is a cash-basis taxpayer, salary is taxable to Burg when

actually or constructively received, whichever is earlier.

you cannot deduct any of the expenses listed if he does not itemized deductions, (business-related unreimbursed expenses for 2018: entertainment, travel, and education)

additionally, unreimbursed employee business expenses are not deductible beginning in 2018

mortgage payments are treated as

alimony

with regard to the alimony deduction in connection with a 2018 divorce,

alimony payments must terminate on the death of the payee spouse. alimony can only be in cash

admission of a new general partner requires approval of

all of the existing partners, both general and limited

a seller can recover the full contract price if the goods have

already been identified to the contract and the seller is unable to resell them at a reasonable price.

a deduction is allowed for the

amount distributed

The IRC provisions dealing with tax return preparation

apply to a CPA who prepares the tax returns of the president of a corporation the CPA audits, without charging he president. this is an example of implied compensation any one may prepare the return, passage of an exam is only necessary to practice before the IRS

a taxpayer must make the contribution to a deductible IRA by what date

april 15th, to be deductible for the current year

an unmarried individual with modified AGI of 25k paid $1000 interest on a qualified education loan entered into on July 1, how may the individual treat the interest for income tax purposes

as a $1000 deduction to arrive at AGI for the year. maximum deduction is limited to $2500 and is reduce by MAGI in excess of $60000 if single, HOH, or qualifying wiwo, 120k if MFJ.

the generation-skipping transfer tax is imposed

at the higher tax rate under the transfer tax rate schedule.

conviction of felony would not result in an

automatic expulsion from the AICPA

the doctrine of substantial performance dictates that the value of the defect

be deducted form the price

acts that will bar a general discharge in bankruptcy 1. debtor tries to hide some property to prevent the estate from getting it 2. the debtor is unwilling to explain satisfactorily why some assets are missing 3. the debtor intentionally destroyed records of assets

beating up a creditor during an argument about the bankruptcy proceedings will not bar a general discharge in bankruptcy. This is an intentional tort and the liability for these injuries would not be discharged in bankruptcy;

a personal holding company is a C corporation in which more than half of the value of its outstanding stock is owned (directly and indirectly) by five or fewer individuals and which received at least 60% of its adjusted ordinary gross income fro passive sources,

because the top corporate tax rates have historically been lower than the top individual tax brackets, some shareholders of closely held corporations have sought to retained earnings within the corporation as a strategy to avoid the higher individual tax rates, to prevent list, congress enacted a penalty tax on certain C corporations-the personal holding company tax.

damages received from patent infringe is included in

book income and taxable income, no adjustment necessary

remedy of specific performance is not available for

breach of a personal service contract

in a like-kind exchange, no gain or loss is recognized on the exchange of

business or investment property for poetry of a like kind(same class of property) unless cash or unlike property was received.

Norwood is an accrual-basis calendar-year corporation, it can elect to deduct a contribution authorized by its board of directors during 2018 so long as the contribution is subsequently paid no later than three and one-half months after year-end (i.e., by April 15th).

but the maximum deduction is limited to (book income + charitable deduction)*10%

unused foreign tax credit

can be carried back one year and forward 10 years.

a general partner of a partnership

can by virtue of his acts, impose tort liability upon the other partners, bc partners are jointly and severally liable for all debts of partnership including torts committed by a copartner while carrying on partnership business.

cash method vs accrual method tax

cannot use cash method if you have inventory or if you're a C corporation. have revenue over 25million. you can use cash method if you're a qualifed personal service corporation, gross receipt of 25million or less of the previous 3 years.

corporation cannot deduct

capital loss when computing taxable income

charitable contributions subject to the 60% limit that are not fully deductible in the year made may be

carried forward 5 years.

a partnership is reduced by a

cash distribution

in a partnership, a liquidating distribution normally has no tax effect for either partner, $0 gain/loss for both partnership and partners

cash received by partner reduce his tax basis, property received by the partner equal his tax basis-cash received.

partnership basis is reduced by

cash received, and than the basis of inventory received. land received would have the remaining dollar of partnership interest, which if sold, would result in capital gain or loss.

no gain is recognized when you buy ownership interests with

cash, you contribute property and own less than 80% of the interest, you must recognize gain

duress

duress is the actual or threatened causing of an action or inaction which, contrary to a party's free will and judgment, forces him/her to enter into a contract. makes a contract voidable

when computing AMT in 2018, the individual taxpayer may take a deduction for which of the following items State income taxes Real property taxes Standard deduction Casualty losses in Federal Disaster Area

casualty losses in federal disaster area

individual who is not eligible to practice before the IRS exclude

certified financial planner, he is not among the list of those who can practice before the IRS by virtue of his certification as a financial planner.

Below the line deductions from AGI

charitable donations medical expenses tax interest expenses

a preparer of a tax return may incur penalties under the IRc in this case

claim a substantial deduction for a loss resulting from an accidental fire

a preparer of a tax return may incur penalities under the IRC in all of the following cases except where the taxpayer

claims a substantial deduction for a loss resulting from an accidental fire.

under section 12 of the 1934 act, in addition to companies whose securities are traded on a national exchange, what class of companies is subject to the SEC's continuous disclosure system

companies with assets in excess of $10 million and 500 or more shareholders.

an independent contractor is typically paid for the

completion of the project rather than on an weekly, hourly or monthly basis

MACRs five year property includes

computer, automobiles, trucks, calculator, copiers, and peripheral equipment

executory contract

contract not fully performed on both sides. eg. you enter an agreement to lease a building from some one. the lessor must provide the building over the term of the lease and the lessee must pay rent.

this is required under the act of 1934 or the SEC's reporting requirements issued pursuant thereto

current reporting by issuers of registered securities of certain specified corporate and financial events within 4 days of occurrence. such as a change in corporate control, revaluation of assets, or a change in the amount of issued securities must be filed within 4 days. quarterly financial reports must be filed, they are not required to be audited.

if you fire your agent, you need to give what kind of notice to your customers?

customers need to have direct, actual notice. otherwise you would be liable

employment-related expenses eligible for the child care tax credit include

daycare and baby sitter, maximum child care tax credit is $3000 for one child two children is $6000

a cash basis taxpayer, borrowed money from a bank and signed a 10-year interest-bearing note on business property on january 1 of the current year, the cash flow from Nan's business enabled Nan to prepay the first three years of interest attributable to the note on dec 31 of the current yea, how should Nan treat the prepayment of interest for tax purposes

deduct the current year's interest and amortize the balance over the next two years. interest expense must be deducted in the period in which it relates. no advance deduction for interest expense is allowed, instead, interest expense must be amortized over the period to which it relates, here, Nan can deduct the current year's interest expenses, but the remainder must be amortized over the following two years.

Partnership paid employee salaries and wages

deductible by the partnership in arriving at partnership ordinary business income, salaries and wages are an ordinary business expense and are deductible by the partnership in arriving at the partners's ordinary business income.

interest forfeiture penalty is treated as

deduction from gross income in arriving at AGI

Under the half-year convention that generally applies to ?

depreciation personal property one-half of the first year's depreciation is allowed in the year in which the property is placed in service, regardless of when the property is placed in service during the year, and a half-year's depreciation is allowed for the year in which the property is disposed of, regardless of when the property is disposed of during the year.

accounting income of a trust includes

dividends, interest from corporate bonds, tax-exempt interest from state bonds excludes; capital gain and trustee fee allocated to corpus (principal)

qualifying relatives do not include cousins and foster parents

do include cousins

does this qualify as unrelated business income? organization sells bred made and sold by volunteers contributed funds are invested and dividends are earned

doesn't qualify

The certificate of limited partnership requires the names of the

general partners, but not the limited partners.

a common law duty is delegated even though the

duty delegated is the payment of money and the delegatee is not of as equal creditworthiness as the delagator. This fact by itself does not mean that the delegation will materially change the risk of the other party, since the payment of money does not result in a materially different performance to the obligee. Recall that the obligee has a right to sue either the delegator or delagatee in the event that payment is not made, so there is no material change in risk.

tax free recapitalization

e.g paying off bond holders with preferred stock that cost more then cash.

this credit can result even if the individual has no income tax liability

earned income tax credit.

under the federal subchapter s revision act, all corporations are designated as

either a subchapter S corporation or a subchapter C corporation

Even though the accountant was not liable for the work in the regular audits, the accountant can be held liable for

either fraud or negligence or both in the S-1 Review whenever the facts are present to support them. This is true even though the S-1 Review is not a full audit.

if someone claimed to be my agent and made a contract in my company's name. I notified the third party of the truth that he is not my agent, the third party may not

enforce the contract even if my company does not wish to be bound: because the agent has no authority to act on my behalf, therefore the contract cannot be enforced MCQ: AGEN-0008B

partial liquidation of the distributing corporation is treated as

entirely as capital gain

t/f The accumulated earnings tax can be charged to a corporation in addition to its regular income tax. This tax is designed to discourage corporations from reducing dividend payments so that the stockholders can defer paying taxes on the distributions. A corporation is required to self-assess and report if any accumulated earnings tax is due.

false, the amount of accumulated earnings is viewed as excessive when it is compared to the reasonable needs of the business, the decision as to the reasonable needs of the business must be made by the government, thus, a company cannot self-assess whether an accumulated earning tax is due.

t/f A corporation may be deemed a personal holding company so that a personal holding company tax is due if over 50 percent of ordinary income for the year is passive (comes from rent, dividends, interest, and the like).

false, a company is viewed as a PHC, if during the last half of the tax year, over half of the stock is owned by 5 or fewer individuals and 60% or more of the company's ordinary income is passive.

t/f A corporation has an office building that is used in the business. For federal income tax purposes, this building is referred to as Section 1245 property.

false, this would be a section 1250 property, includes business land and most depreciable real business property such as an office building or warehouse.

no loss deduction is allowed on the sale or exchange of property between members of a family

family includes brothers, sisters, half-brothers half sisters, spouses ancestors and lineal descendants (children, grandchildren, etc.). Since in-laws and uncles are excluded from this definition of a family, a loss resulting from a sale or exchange with an uncle or between in-laws would be recognized.

at family partnership, services performed by family members must be

first reasonably compensated, deducted from ordinary income before income is allocated between partners according to capital interests.

a taxpayer will be subject to an underpayment of estimated tax penalty if the taxpayer did not pay enough tax either through withholding or by estimated tax payments

for 2018, there will be no penalty if the total tax shown on the return less the amount paid through withholding is less than $1000 additionally, individuals will incur penalty if the amount of tax withheld plus estimated payments are at least equal to the lesser of 1) 90% of the current year tax 2) 100% of the prior year's tax

no gain or loss is recognized if stock is exchanged solely

for stock in a corporation that is a party to the reorganization, stock received is the same as the basis in the stock transferred.

use the cash method of reporting, an individual should report gross income

for the year in which income is either actually or constructively received in either cash or in property.

this is the form you use to apply for tax exempt charitable organization status

form 1023

although both form 10k and form 10q must be filed with the SEC, only form 10-k need to be audited by independent public accountants

form 10q just need to be reviewed

this is the form you file every year to maintain your status as charitable organization

form 990

One-half of a self-employed taxpayer's self-employment tax is deductible

from the gross income in arriving at AGI, foreign real estate taxes, foreign income taxes personal property taxes can be deducted as itemized deductions from AGI

filing a valid petition in bankruptcy acts as an automatic stay of actions to

garnish the debtor's wages: yes collect alimony from the debtor: no when a valid petition in bankruptcy is filed, it acts as an automatic stay which stops the collection of most debts and the enforcement of most legal proceedings against the debtor's estate, but an automatic stay is not effective to prevent the collection of alimony.

At-Risk Limits.

generally, your deductions cannot exceed the amount you have at risk, roughly, an amount at risk is an amount you invested and could lose, an amount not at risk exists when there is a part of your investment basis that you are protected from losing. The amount you have at risk is like basis in that you cannot deduct losses in excess of your at risk amount

If a CPA recklessly departs from the standards of due care when conducting an audit, the CPA will be liable to third parties who are unknown to the CPA based on:

gross negligence: this is difference from ordinary negligence which is failure to comply with the standards of due care while conducting an audit. a CPA is not liable to foreseeable 3parties for ordinary negligence.

life insurance premium paid reduces your basis in the partnership

guaranteed payment to the other partner has not effect on your basis in the partnership. This would already be deducted in the partnership's computation of ordinary business income and is not separately taken into account in determining Mack's basis for the partnership interest.

guaranteed payment is a payment for services performed by the partners, similar to salary and receive similar income tax treatment

guaranteed payments are either deductible or capitalized by the entity depending on the nature of the payment. the guaranteed payment is deductible by the partnership and subject to self-employment tax to the receiving partner

a general partnership must

have two or more partners

a minor can disaffirm a contract at any time until he reaches the age of majority or a reasonable time thereafter

heavily protected, dont enter contract with a minor

the common carrier is permitted to limit its liability to a specified dollar amount,

however, it would not be liable at all if the loss was due to act of the shipper or accidents deemed acts of god

when a purchaser of property assumes a mortgage note, he promises the mortgagor he will pay the mortgage debt,

however, the mortgagor is not relieved of his own duty to pay the debt, but the mortgagor does have a right to insist that the purchaser make payment, the mortgagee can look to either party for payment the mortgagor on the basis of the original mortgage agreement, or the purchaser on the basis of being a 3rd party beneficiary of the assumption agreement, the mortagee can sue either party for payment.

under the sales article of the UCC, unless a contract provides otherwise, before tile to goods can pass from a seller to a buyer, the goods must be

identified to the contract. seller can keep possession of goods and identify them to the contract and still have title pass to the buyer

an s corporation must have 100 or fewer shareholders who are individuals(other than nonresident aliens), certain trusts, or estates (including bankruptcy estates)

if a corporation has been a S corporation since inception, it can have both passive and nonpassive income and be owned by a bankruptcy estate, non resident aliens may not own shares of S corporation

an involuntary petition in bankruptcy

if not contested will result in the entry of an order for relief by the bankruptcy judge. Only if the petition is contested will the creditor(s) be required to prove either that the debtor is not paying her/his debts as they mature, or that during the 120 days preceding the filing of a petition, a custodian was appointed or took possession of the debtor's property.

surety general rule is that a surety is released from liability for acts of the creditors which materially increased the surety's risk

if the creditor failed to give notice of prior embezzlement, the surety's risk is materially increased and the surety would not be liable

life insurance proceeds paid by reason of death are excluded from income if paid in a lump sum or in installments.

if the payments are received in installments, the principal amount of the policy divided by the number of the payment is excluded each year

passive activity losses are generally only deductible against passive activity income

if there is a net passive loss and multiple loss activities, the suspended net passive loss must be allocated to loss activities in proportion to the amount of loss generated by each activity. eg. activity x and y generated $80000 of passive losses, the amount of the $60000 suspended loss allocated to activity would be (30/80)*60=22.5

losses from passive activities can only be used to offset income from passive activities,

if there is not enough passive activity income to absorb passive activity losses, the unused losses are carried forward indefinitely or until the property is disposed of in a taxable transaction

when you oppose the involuntary petition to file chapter 7 bankruptcy, you will fail if you

if you don't pay your debts as they become due. 2( )you will also fail if a receiver was appointed to take control of the debtor's property within 120 days prior to the filing of involuntary petition

in a distribution when a corporation is liquidated, the corporation handles the distribution as if the property has been sold for its FMV, in that way, income tax on any appreciation is not avoided

in a corporate liquidation, if the owner held 80% or more of the corporation stock, no gain or loss is recognized by either party

this is false regarding a chapter 13 a bankruptcy , individuals in general need not have regular income

in general, individuals need to have regular income along with other specified requirements in order to file for chapter 13 bankruptcy

under the cash method of tax accounting, income is reported when first actually received or constructively received either in cash or in property

in the case of property, the FMV of the property received is the amount to be reported. constructive receipt means that an item of income is unqualifedly available without restriction (interest resulting from a savings account is reported as income for the year in which the interest is credited to the account)

the holding period of property acquired by a partnership as a contribution to the contributing partner's capital account

includes the period during which the property was held by the contributing partner.

the corporation's holding period in the property always

includes the period that the transferor held the property before the exchange regardless of the character of the property to the transferor

registration under 1934 act has its purpose to provide potential investors with full and fair disclosures of all material information relating to the issuance of securities

including information as to how the proceeds will be used.

a partnership functions as a pass-through entity and its items of income and deduction are passed through partners on the last day of the partnership's taxable year,

income and deduction items pass through to be reported by partners even though not actually distributed during the year.

IRD (Income in Respect of a Decedent)

income that was owed to a decedent at the time he or she died. ignore apartment building, IRD on apartment building and boat.

James is a 20 percent partner in the Portland Partnership. James has a tax basis (capital share) in the partnership of $36,000. This year, the partnership reported revenues of $200,000, cost of goods sold of $120,000, and a guaranteed payment to James for his work of $30,000. There were no other revenues and expenses. By what amount will these events increase the taxable income reported by James this year?

increase in deduction $40000 200000-120000-30000=50000 50000*0.2=10000 10000+30000=40000

if property acquired by gift is sold at t again, its basis is the donor's basis,

increased by any gift tax paid attributable to the net appreciation in value of the gift

rental loss deduction may be carried forward for how long?

indefinitely

under the liquidation provision of chapter 7 of federal bankruptcy code, certain property acquired by the debtor after the filing of the petition becomes part of the bankruptcy estate, an example of such property is

inheritances received by the debtor within 180 days after filing of the petition

insiders and tippee

insiders eg. CEO and CFO tippee: an old friend who acquired insider information through CEO

under the RMBCA a stockholder has the right to

inspect the books and records with 5 days notice

constructive fraud elements: 1. misrepresentation of material fact 2. reckless disregard for truth 3. reasonable reliance by the injured party 4. injury

intent does not need to be shown as in the case of actual fraud.

the assignee is entitle to the assigning partner's

interest in partnership profits and surplus on the dissolution of the partnership

investment income equal

interest income+ qualified dividend income+ long term capital gain investment interest expense is limited in its deduction to the lesser of net investment income or actual investment interest paid.

interest payments from business loans and other forms of credit acquired to fund business expenses are ordinary tax deductions,

interest paid or accrued during the tax year are fully tax deductible.

Reg CF requires investors to invest through

intermediaries,(they are required to invest through brokers or funding portals that can do some degree of investigation regarding offerors.) Reg CF has no purchaser qualifications must file an offering statement with the the SEC with information that are required to be disclosed. The ceiling began at $1 million and is periodically adjusted for inflation.

separately passed through items for an S corporation includes

investment interest expense a net section 1231 loss foreign income taxes

donee beneficiary

involves a gift

Undue Influence

involves the forcing of a party to enter into a contract by taking unfair advantage of a relationship of trust between the two parties or by taking unfair advantage of another party's weakness of mind.

Option Offer (irrevocable for the stated time period)

is defined as "a promise which meets the requirements for the formation of a contract and limits the promisor's power to revoke an offer." An option contract is a type of contract that protects an offeree from an offeror's ability to revoke their offer to engage in a contract.

rental loss deduction is 25000

is reduced by 50% of the taxpayer's AGI over $100000 the entire deduction is phased out at $150,000 of AGI

the trustee in a bankruptcy proceeding

is the rep of the bankrupt's estate and as such has the capacity to sue and be sued on its behalf.

liquidated damages are for clause in the contract

liquidated damages set the amount of the compensatory damages for the type of breach that occurred. in other words, liquidated damages are those agreed to in a contract to set the amount of damages in the event of breach of contract

for income tax purposes, the estate's initial taxable period for a decedent who died on October 24

maybe either the calendar year, or a fiscal year beginning on the date of the decedent's death.

this parties generally has the most management rights

member of a LLC, he has the rights of a general partners

the owners of a LLC are called

members

type E reorganization

mere change in identity, form or place of organization

office equipment purchased and placed into service during 2018, what convention should you use

mid quarter

how many public company audits per year does a CPA firm that is registered with the PCAOB have to perform before it receives an annual inspection from the PCAOB

more than 100 audits

for a decedent dying in 2018, a federal estate tax return

must be filled if the decedent's gross estate is greater than $11,180,000

provision for federal income tax and interest paid on loan to purchase municipal bonds

must be included in taxable income

to qualify as an exempt organization, the applicant

must not be a private foundation organized and operated exclusively to influence legislation pertaining to protection of the environment an organization must be one of those classes upon which exemption is specifically conferred by the IRC

PMSI in tables used in restaurants PMSI in tables used by retailers to sell to consumers

neither of these are consumers goods but instead in equipment or in inventory. attachment alone is not enough to perfect the security interest

does purchase of land for investment affect your basis in partnership?

no, The partnership's purchase of land does not affect the partners' basis. The partnership's purchase of land is an* asset purchase *that does not affect the partners' basis or the partnership's income.

can you claim your son as a dependent in this situation? age 21, full time student, had $6200 income and provided 70% of his own support?

no, bc he provided over 1/2 of his own support,. a dependent generally receive over 1/2 of support from you and have less than $4150 of income.

are theft losses deductible?

no, beginning 2018, theft losses are not deductible for individuals because the are not attributable to a federal declared disaster

are rental income and distribution to partners included on computation of partner's ordinary business income

no, rental income pass through directly to the tax returns of the individual partner (other items includes dividend income, interest income, charitable contributions, and capital gains and losses) distributions to partners are not viewed as having any impact on ordinary business income.

does increase/decrease recourse debt impact the partnership basis

no, the members of an LLC are not personally liable for the entity's debts, thus the recourse debt on the balance sheet does not impact the bases that the LLC members have in their LLC interests.

legal fee in connection with production of income: is this deductible?

no, this is expenditure incurred in the production of income, which is 2% misc itemized deductions that are not deductible beginning 2018

child and dependent care credit is

nonrefundable and may vary from 20% to 35% of the amount paid for qualifying household and dependent care expenses incurred to enable a taxpayer to be gainfully employed or look for work

allowable deductions for AMT includes medical expenses: limited to amount in excess of 7.5% of AGI qualified mortgage interest charitable contributions

not deductible personal, state and local taxes for the home mortgage interest if the loan proceeds were not used to buy, build, or improve the home.

the municipal bond interest is tax-exempt and

not included in gross income in your tax return

a stockholder's right to inspect books and records of a corporation will be properly denied if the purpose of the inspection is to

obtain stockholder names for a retail mailing list. the shareholders' right may be denied if the corporation can show that the shareholder's motives is for an unwarranted purpose, for a purpose hostile to the corporation (to engage in competing business), or to satisfy idle curiosity.

a distributee partner can recognize loss only upon the complete liquidation of the partners's interest receipt

of only money, unrealized receivables or inventory

MACRS 7 year property includes

office furniture, fixtures, and equipment, as well as agricultural machinery and equipment. and computer desks

under the bankruptcy code, a trustee has the power to set aside preferential transfers made by the debtor to creditors, preferential transfers are those made for antecedent debts which enable the creditors to receive more than he would have otherwise received under the liquidation proceedings,

one exception to the trustee's power to avoid preferential transfers is when a security interest is given by the debtor to acquire property that is perfected within 10 days after such security interest attaches, this is called an enabling loan, enabling loan cannot be set aside by the trustee.

which one is deductible when computing AMT for individuals? 1State income taxes 2Home equity mortgage interest when the loan proceeds were used to purchase an auto 3Real estate taxes 4 Home equity mortgage interest when the loan proceeds were used to add an additional room to the house

only the mortgage interest where the loan proceeds were used to improve the land is deductible

in order to satisfy the UCC statute of frauds regarding sale of goods,

only the quantity term must be included in the writing, failure to include the time of payment willnot cause the writing to fail bc a reasonable time of payment will be inferred failure to include other terms of the contract, including price, will not result in an insufficient writing either, the code implies that the parties will in good faith determine a reasonable price

the mail box rule, an acceptance is effective when sent if transmitted by the means authorized by the offeror

or by the same means used to transmit the offer if no means was authorized. but if offeror may say that acceptance is effective only when received by the offeror

sale of inventory results in

ordinary income

the value of partnership interest received must be reported as

ordinary income equal to the FMV

partnership paid fines and penalties

partners are not entitled to a deduction and decrease their basis in the partnership partners are not entitled to a deduction and decrease their basis in the partnership,

partners made a cash distribution to partner C

partners do not include the cash as income, but must reduce their basis in the partnership, in a non-liquidating distribution normally the partner should recognize no gain or loss on the distribution of money or other property by the partnership

partnership received municipal bond interest

partners do not include the income, but increase their basis in the partnership

when partnership's business and financial operations are discontinued, and there is no longer at least two partners

partnership is terminated. Before 2018, a technical termination occurred if within a 12-month period there was a sale or exchange of 50% or more of the total interest in partnership capital and profits. The technical termination rules no longer apply.

this must take place for a corporation to be voluntarily dissolved

passage by the board of directors of a resolution to dissolve

real estate rental passive loss can be used to offset the

passive income from the S corporation

the rule limiting the deductibility of passive activity losses and credits applies to

personal service corporations. also to individuals, estates, trusts, closely held c corporations. intended to prevent tax payer from sheltering personal service income by creating personal service corporations and acquiring passive activities losses at the corporate level.

unemployed compensation is generally available to only

persons unemployed through no fault of his own.

generally, no gain or loss is recognized on the the transfer of

property to a partnership in exchange for a partnership interest. since Carr's gain is not recognized, there will be a carryover basis of $30000 for the stock to the partnership, and Carr will have a $30000 basis for the 25% partnership interest received.

are these items deductible in current year taxable income?

provision for federal income taxes: not deductible net long-term capital loss: not deductible keyman life insurance premium( corporation is beneficiary) : not deductible not deductible meaning they will all be added to net income per book to calculate taxable income

does this qualify as unrelated business income? sale of appliances by a senior citizens center to its member

qualifies

gross negligence

reckless depart from standards, will make CPA liable to 3parties

this shareholder right is enforceable by means of a derivative suit

recovering damages to the corporation from a third party

child care credit is 35% of qualified expenses

reduce by one percentage for every 2000 over AGI 15000, down to minimum of 20%

partnership sold equipment to partner A at a $6000 loss

related rules result in current nonrecognition of transaction

this corporation action is subject to shareholder approval

removal of directors board of directors generally may take this action without shareholder approval

Generally, gains and losses on intercompany transactions during consolidated return years are deferred and

reported in subsequent years when a restoration event occurs

the antifraud provisions of rule 10b-5 of the 1934 act

require that the wrongful act must be accomplished through the mail, any other use of interstate commerce, or through a national securities exchange

this statement applies to a sale to approval under the UCC sales article

risk of loss for the goods passes to the buyer when the goods are accepted after the trial period. the seller retains the title and the risk of loss until the buyer accepts the goods.

statute of frauds $500 limit applies to

sale of good, not for the service contract.

individual income taxes-partnership income interest expense on taxable investment debt

schedule A-itemized deductions if money is borrowed to earn taxable investment income then that income is taxable, the related interest expense on the debt can be deducted as an itemized deduction

individual income taxes-partnership income interest income from US treasury bond

schedule B-interest and ordinary dividends pass through and is reported as interest income by the individuals

individual income taxes-partnership income short term capital loss

schedule D-capital gains and losses, passes through and is reported on schedule E partnership and royalty income

ordinary income of partnership is reported

schedule E- supplemental income and loss all remaining income and expenses after the pass through items have been reported are netted together and reported as partnership income on schedule E

when more than 5 million of securities are being offered , an exemption from the 1933 act is available under rule 506 of reg D.

securities under this act include debentures and investment contracts.

gross income includes $30,000 of current rents, $10,000 of advance rents, and $15,000 lease cancellation payments. Advance rents must be included in gross income when received regardless of the period covered or the accounting method used.

security deposits held in another account will be included in gross income at a later date if not returned to tenants

the personal holding company tax should be

self-assessed by filing a separate schedule with a regular tax return, PHC tax can only be imposed on corporations

this allowable deduction can be claimed in arriving at an individual's AGI

self-employed health insurance deduction

guaranteed payments to partners are deductible in computing partnership ordinary income, there would be no NOl deduction, since prior losses would have already been passed through to partners,

since charitable contributions are subject to percentage limitation, they must be separately passed through to partners and cannot be included in computing ordinary income, a deduction for a partner's personal exemptions would be allowed on the partner's tax return, not on the partnership return.

A couple filed a joint return in prior tax years. During the current tax year, one spouse died. The couple has no dependent children. What is the filing status available to the surviving spouse for the first subsequent tax year?

single, Since the couple was married at the date of death of one spouse, a joint return can be filed for tax year during which the spouse died. However, since the couple had no dependent children and assuming that the surviving spouse did not remarry, the only filing status available for the first year subsequent to the tax year in which the spouse died would be that of a single taxpayer.

to determine corporation's total tax liability for the year, Personal holding companies cannot be subject the accumulated earnings tax,

so if both penalty taxes could apply, only the personal holding company tax must be paid. not both

profit sharing in a partnership is

split equally unless stated otherwise in the partnership agreement.

state income tax and self-employment tax, which one is deductible for taxes in the itemized deductions for schedule A

state income tax is deductible self-employment tax is not deductible as an itemized deduction, instead, a portion of the self-employment tax is deductible from gross income in arriving at AGI.

DRD less than 20%->> 50% deduction 20% or more but less than 80%->> 65% deduction 80% or more->> 100% deduction

steps to calculate DRD 1. multiply the dividends received by the deduction percentage 2. multiply the taxable income by the deduction percentage 3. the deduction is the lesser of step 1 or 2, unless the taxable income is less than DRD, then step 1 amount is used.

this is not a corporate reorganization as defined in the IRC

stock redemption

the filing of an involuntary petition in bankrupcy

stops the enforcement of a judgment line against property in the bankruptcy estate., ] it also stops the enforcement of most collections of debts and legal proceedings against the debtor's estate.

how is depreciation deduction for nonresidential real property, placed in service in 2018, determined for regular tax purposed using MACRS

straight line method over 39 years

an intended donee beneficiary has the rights to

sue in the even of breach.

under the UCC, this action will best perfect a security interest in a negotiable instrument against any other party

taking possession of the instrument,. a holder can become a holder in due course even if a security agreement is filed.

interest income from US treasury bonds is

taxable

the charitable contribution base for a corporation is its

taxable income computed before the charitable contribution deduction, the dividends-received deduction and a capital loss carryback (but after capital loss carryforward)

George Granger sold a plot of land to Albert King on July 1, 2018. Granger had not paid any realty taxes on the land since 2016. Delinquent 2017 taxes amounted to $600, and 2018 taxes amounted to $700. King paid the 2017 and 2018 taxes in full in 2018, when he bought the land. What portion of the $1,300 is deductible by King in 2018?

the 600 delinquent taxes charged to the seller and paid by king are not deductible, but are added to the cost of property, the $700 of taxes for 2018 are apportioned between the seller and the king according the number of days that each held the property during the year, king's deduction would be (184/365)*700=$353

Beginning in 2018, net unearned income in excess of $2,100 is taxed at

the Trust and estate return rates due to the Kiddie tax rules for children 18 and under.

net long-term capital loss from the sale of marketable securities

the a reconciling item from book item to taxable income. deducted per books, but is not deductible in computing taxable income.

in the instances in which the offeree sends a rejection first and then an acceptance,

the acceptance is not effective unitl received,

An S corportion may deduct

the amortization of organization expenditure.

previous unrecaptured sec 1231 losses can be use to reduce

the amount of LTCG.

the bad debt deduction should be consist of

the amount of actual bad debt expense written off

donation of appreciated stock held more than 12 months is a contribution of intangible, long term capital gain appreciated property

the amount of contribution of the stock's FMV of $70000, but is limited in deductibility for 2018 to 30% of AGI. the amount of contribution in excess of 30% limitation can be carried forward for up to five years, subject to the 30% limitation i the carryforward years.

provision for income taxes

the amount of income tax expense for a given period (estimated amount that you're expected to pay for the current year)

a cash basis taxpayer generally reports income when received,

the amount of income to be reported is the amount of money, plus the FMV of other property received.

this provision of 1934 act applies despite the fact that corporation's securities are exempt from registration

the antifraud provisions. no need to file reports no proxy provisions no provisions imposing internal accounting controls

in a like-kind exchange of property held of investment, a realized gain will be recognized only to the extent of unlike property received ( cash plus maybe boat)

the basis of the acquired like-kind property reflects the deferred gain resulting from the like-kind exchange, and is equal to the basis of the property transferred, increased by the amount of gain recognized, and decreased by the amount boot received.

Pine has a security interest in certain goods purchased by Byron on an installment contract. Byron has defaulted on the payments, resulting in Pine's taking possession of the collateral. Which of the following is correct?

the collateral may be sold by Pine at a private sale and, if the collateral is consumer goods, without notice to other secured parties. Pine only has the option of selling the collateral if the debtor has paid 60% of the cash price on a PMSI on consumer goods, not business equipment

a 20% credit is allowed for qualifying expenditures made to rehabilitate a certified historical structure

the credit is claimed ratably over five years beginning with the year the building is placed in service.

this event will reduce a surety's liability to the creditor

the creditor failed to notify the surety of a partial surrender of the principal debtor's collateral

the creditor's possession of documents of title (ie, the negotiable warehouse receipts) means

the creditor has a security interest in the goods which has attached and is perfected. thus a perfected security interest in the warehouse receipts (by taking possession of the documents) is also a perfected security interest in the goods covered by the document the creditor does not have to file a financing statement, the the filing would not protect the creditor against a good-faith purchaser to whom the warehouse receipts have been duly negotiated.

in order for a security interest in goods to attach

the creditor must have given value. one of three conditions 1. the debtor has rights int he collateral 2. the creditor extends value 3. a record of the security agreement exists.

under the UCC secured transaction article for a security interest to attach

the debtor must agree to the creation of the security intest,. the secured party must also give value and the debtor must have rights in the collateral filing a financing statement is one of the methods used to perfect a security interest but is not required for attachment.

this action is required to ensure the validity of a contract between a corporation and a director of the corporation

the director must disclose the interest to the independent members of the boards and refrain from voting shareholders do not have to review and ratify the contract.

an S selection will be effective as of the first day of a taxable year if

the election is made on or before the 15th day of the 3rd month of the taxable year. e.g elected in feb 10, is effective as of jan 1 2018

when the grantor of a trust retains substantial control over the trust, such as the power to revoke the trust or a discretionary power to have trust income distributed to the grantor or grantor's spouse

the income from the trust will be taxable to the grantor.

type E reorganization (recapitalization)

the issuance of preferred stock in exchange for bonds, if no cash was exchanged, no part of realized gain is recognized.

in a LLC, unlike the common corporation, the interests of the members are not freely transferable,

the other members have to agree to admit new members.

the entire amount of precontribution gain would be distributed to who?

the partner who contributed the property,

a partner can deduct the ownership % of any partnership ordinary business loss but only up to

the person's at risk balance, the at risk balance is the tax basis in the partnership plus any any loan to the partnership plus the ownership's share of the debts of the business.

in a complete liquidation of a partner's interest in a partnership,

the property distributed will have a basis equal to the adjusted basis of the partnership interest reduced by any money received in the same distribution.

double-weighted method

the property factor would be counted twice and then the total would be divided by 4 to determine the average, e.g the apportionment factor would then be (60%+50%+79+49%+49%)/4=52%

Schedule M-1 on Form 1120 is for ?

the reconciliation of net income per the company's books to the amount of taxable income being report. provides an analysis of the change in the corporation's unappropriated retained earnings.

under the tax benefit rules, if the taxpayer takes a deduction in a previous year and then receives a refund or reimbursement in a later year

the refund/reimbursement must be included in income to the extent of the benefit of the deduction received.

when the seller of the goods is a merchant seller,

the risk of loss does not pass until the buyer takes receipts of the goods

the real estate tax deduction is apportioned between

the seller and the buyer

one of the steps of merger is that

the stockholders need to be given a notice of the merger plan

to be eligible for American opportunity credit

the student must be enrolled full-time for at least one academic period during the year. the credit is available on a per student basis and covers tuition paid for the taxpayer, spouse and dependent. to be eligible, the student must be enrolled on at least a half-time basis for one academic period during the year.

if a tax appeal is deemed frivolous,

the tax payer will be given 30 days to withdraw or amend the CDP appeal.

this disqualified an individual from the earned income credit

the taxpayer has a filing status of married filing separately earned income credit is a refundable tax credit for eligible low income taxpayer, to be eligible, you must have earned income and generally must maintain a household for more than half the year for a qualifying child. a qualifying child includes your child or grandchild who lives with you for more than half of the year, and is under age 19, or a full time student under age 24, or permanently or totally disabled. MCQ: ITAX-0154

if stearn was not an agent of the lehman corporation when the contract was made because there must be a principal to have an agent and lehman had not been formed yet

then stearn would be personally liable for making contract

when the contracts an agent is entering into for the principals are required to be in writing,

then the agent's authority to do should also be in writing.

partnership guaranteed payments do not increase your basis in the partnership bc

they are already reflected in the calculation of the partnership's income/loss

partnership and s corporation and net operating loss deduction

they are not entitled, because they are pass through entities whose expenses and losses pass through to be reported on the tax returns filed by their owners.

liquidated damages become a penalty, which is void when

they cross into penalty territory when the penalty is equal to or more than the actual contract cost

issuers of securities which are traded over-the-counter must report only when

they have in excess of $10 million in total assets and more than 500 holders of record (stockholders) as of the last day of the issuer's fiscal year.

if neither husband and wife participate in a qualified retirement plan. they are eligible to make deductible contributions to their IRAs

they may contribute and deduct a total of $11000 to their individual retirement accounts.

is this a syndication cost or organizational cost? accounting fees to prepare the representations in offering materials?

this is a syndication cost, which include the costs connected with the issuing and marketing of partnership interests such as commissions, professional fees, and printing costs, these costs must be capitalized and can neither be amortized or depreciated.

Although there is typically a contract between the principal and agent in an agency relationship,

this is not required.

a corporation generally recognizes gain on the distribution of appreciated property in the process of liquidation,

thus the corp must recognize gain on the distribution of the notes to the extent that the FMV of the notes exceeds the basis of the notes.

what is the purpose of cumulative voting

to allow minority shareholders to gain representation on the board of directors. Cumulative voting gives the shareholder one vote for each share owned times the number of directors being elected. This gives minority shareholders an opportunity to get some representation on the board by voting all shares for one or two directors.

a corporation generally must pay for installments of estimated tax, each equal to 25% of its required annual payment,

to avoid the penalty for underpayment of estimated taxes, edge could compute its first quarter estimated income tax payment using the annualized income method. the preceding year's tax liability cannot be use to determine estimated payments if no tax liability existed in the preceding year or a short-period tax return was filed for the preceding year

Sec 351

transfer to corporation controlled by tranferee (non taxable)

Partnership incurred interest expense on partnership investment indebtedness

treated as a separately stated deduction by the partnership and potentially deductible by the partners the deduction for investment interest is limited to the taxpayer's net investment income. since deduction limited to taxpayer's net investment income, one partner may be able to deduct their share of the partner's investment interest expense, while another partner is unable to deduct their share of the same partnership's investment interest expense, investment interest expense is a separately stated item and potentially deductible by the partnership.

partnership received dividends from domestic corporations

treated as separately stated income by the partnership, taxable to the partner

partnership sold an investment held for more than one year at a gain

treated as separately stated income by the partnership, taxable to the partner

T/F In Year One, a company has taxable income of $300,000 and pays $90,000 in income taxes based on a 30 percent effective tax rate. In Year Two, the company has taxable income of $340,000 earned evenly throughout the year. The tax rate is again 30 percent. If this company pays estimated tax payments of $90,000 evenly throughout Year Two, it will avoid a penalty for underpayment of estimated taxes.

true, bc the company had less than 1million in taxable income, the penalty can be avoided by paying 100 percent of last year's tax (90000) or 100% of the current year tax (102000) which ever is less, since the income is earned evenly over the year, 25% must be paid each quarter

guaranty must be in writing, a creditor must first

try to collect from the debtor

3 types of reorganization

type a: statutory merger type e: recapitalization type f: a mere change in identity, form or place of organization

what appear on schedule m2, of form 1120

unappropriated RE: 500 add net income of the year: 204 less contingency reserve: -112 less cash dividends: -30 ending balance: 562

cousins cannot be claimed as dependent unless they live with you for the entire year

uncles on the otherhand, can be claimed as a dependent if you provide more than half of support, he has no income, nor did he file an tax return, is a us citizen.

gifts to spouse limit for deduction is

unlimited, taxfree

PMSI in consumer goods is automatically perfected

upon attachment of that interest.

adams owns a second residence that is used for both personal and rental purposes, during the current year, adams used the second residence for 50 days and rented the residence for 200 days, this statement is correct

utilities and maintenance on the property must be divided between personal and rental use.

tax deficiency paid and tax withheld are

utilize as state and local income taxes in calculating itemized deductions for your current year federal tax return

this is not included in determining the total support of a dependent( life insurance premiums paid on behalf of the dependent)

what is included are clothing tuition payments birthday gifts

one of the elements needed to establish fraud is a misrepresentation of a material fact,

when I told you that you will love the business, this is sales talk and at best a prediction, not a fact, my statement that the business will make at least $150k is a prediction, not a fact, therefore if you only made 75k, I would not be liable bc none of my statements constitute actionable fraud (when a person falsely represent themselves for the purpose of swindling another for money)

a special rule applies if a decedent acquires appreciated property as a gift within one year and death and this property passes back to the donor or the donor's spouse

when this occurs, the beneficiary's basis is the basis of the property in the hands of the decedent before death, rather than FMV at the date of death.

does Life insurance premiums paid on lives of partners (partnership is beneficiary) affect your basis in partnership?

yes, Life insurance premiums on the lives of the partners in which the partnership is the beneficiary are not allowed as a deduction for tax purposes but would be reflected as a book deduction. Even though not a partnership tax deduction, it still decreases Smith's tax basis.

does donation of used clothes to charity count as charitable contribution?

yes, donation of used clothes are deductible for charitable contribution

is liabilities from breaches of contract, dischargeble by bankruptcy?

yes, liabilities from ordinary negligence or from breaches of contract, whether intentional or not, are dischargeble in bankruptcy.

does long term capital loss decrease your basis in a partnership?

yes, ordinary income increases your basis and long term capital loss decreases your basis

does $40000 to a university for a cousin 's room and board require the donor to file a gift tax return

yes? a gift tax return must be filed by a donor if the donor makes a taxable gift (e.g., a gift of a future interest, or a gift of a present interest that exceeds the amount of annual exclusion [$15,000 for 2018]). In determining the amount of taxable gifts, there is an unlimited exclusion that is available for amounts paid on behalf of a donee to an educational organization for tuition, as well as for amounts paid on behalf of a donee to medical care providers for medical services. Thus, the $30,000 to a university for a spouse's tuition, $50,000 to a hospital for a parent's medical expenses and $80,000 to a physician for a friend's surgery would be fully excluded and would not require the filing of a gift tax return. In contrast, the gift of $40,000 to a university for a cousin's room and board would require the donor to file a gift tax return. That is because the $40,000 payment is a gift of a present interest in excess of the annual exclusion, and does not qualify for the unlimited exclusion because it is not a payment of tuition.

to be qualified for child tax credit

you must be under the age of 13, it is $3000 for one child and $6000 for more than one qualifying individual note that qualifying expenses to actual amount paid and the wages earned of the lowest paid spouse credit is qualifying expenses x 20%

interest on state government obligations is tax-exempt income and

you should not include interest income from these obligations on their income tax return.

if you bought furniture in the ordinary course of business

you take free of security interest even if it was perfected, this is true whether you bought the furniture for consumer or business use and whether they knew of the security agreement or not


संबंधित स्टडी सेट्स

finc 420 exam 3 conceptual questions

View Set

Level 1 (Licensed to Drive) - Texas Instructor Taught Drivers Ed

View Set

Contemporary Topics in Math: Test 1

View Set

U.S. History Chapter 11 Section 2 Vocab

View Set

Unit 1: Ancient Civilizations I - Test: Ancient Civilizations I

View Set

Starting Out with Python, 3e Ch 7

View Set

Chapter 26. Bipolar and Related Disorders

View Set

Microeconomics Pre-test Questions

View Set